Bra-Ket Notation, Wave Equation, Particle States

In summary, according to the lecture notes, a particle is in the state |\psi \rangle = \frac{1}{{\sqrt 3 }}|U\rangle + \frac{{a\sqrt {(2)} }}{{\sqrt {(3)} }}i|D\rangle . The up state |U\rangle = \left( {\begin{array}{*{20}{c}}1\\0\end{array}} \right) and the down state |D\rangle = \left( {\begin{array}{*{20}{c}}0\\1\end{array}} \right) correspond to the z
  • #1
squelch
Gold Member
57
1

Homework Statement



A particle is in the state [itex]|\psi \rangle = \frac{1}{{\sqrt 3 }}|U\rangle + \frac{{a\sqrt {(2)} }}{{\sqrt {(3)} }}i|D\rangle [/itex]. The up state [itex]|U\rangle = \left( {\begin{array}{*{20}{c}}
1\\
0
\end{array}} \right)[/itex] and the down state [itex]|D\rangle = \left( {\begin{array}{*{20}{c}}
0\\
1
\end{array}} \right)[/itex] correspond to the z-basis vectors. Detirmine:

a. The value of "a" such that the state is normalized.
b. [itex]\langle U|D\rangle [/itex]
c. The probability of measuring down. [itex]|\langle D|\psi \rangle {|^2}[/itex]
d. The duel vector [itex]\langle \psi |[/itex]
e. The probability amplitude of measuring up.
f. Write out the state [itex]|\psi \rangle [/itex] in terms of right [itex]|R\rangle [/itex] and left [itex]|L\rangle [/itex].
g. What is the probability of measuring right?
h. A measurement is made on our initial [itex]|\psi \rangle [/itex] with our apparatus oriented in the x-direction and [itex]|R\rangle [/itex] was the outcome. Determine the probability of measuring [itex]|U\rangle [/itex] if our apparatus is oriented back in the z-direction.

Homework Equations



None, but as a reference I am using http://ocw.mit.edu/courses/physics/...all-2013/lecture-notes/MIT8_05F13_Chap_04.pdf to guide me through the notation.

The Attempt at a Solution



I'm going to attempt to break this question down best I can as far as I can understand it, but I'm very uncertain (lol) on the details of what I'm doing. I don't want to do cargo-cult physics.

a. I understand that this is the value of a for while the probability [itex]\langle \psi |\psi \rangle = 1[/itex]. It appears that to get there, a should simply equal 1, but it appears there's a possibility that a= 1/i. So, right now my answer is a=1, because I sort of understand the wave function is necessarily a complex function.

b. The probability of measuring up when the state is down should be 0 (zero).

c. It seems the probability of measuring down is a*(2/3) ... or, normalized, just 2/3.

d. The duel vector [itex]\langle \psi |[/itex] is the complex conjugate of [itex]|\psi \rangle [/itex]. So, I might say that [itex]\langle \psi | = \frac{1}{{\sqrt 3 }}|U\rangle - \frac{{a\sqrt 2 }}{{\sqrt 3 }}i|D\rangle [/itex]

e. As a guess, I'd say [itex]\frac{1}{{\sqrt 3 }}[/itex], the coefficient on [itex]|U\rangle[/itex]

f. I feel like I have to make an assumption here: that it's fifty-fifty either way. If so, I might write: [itex]|\psi \rangle = \frac{1}{{\sqrt 2 }}|R\rangle - \frac{1}{{\sqrt 2 }}i|L\rangle [/itex]

g. From that state equation, 1/2.

h. The previous state is lost, so it becomes 1/3.
 
Physics news on Phys.org
  • #2
squelch said:
a. I understand that this is the value of a for while the probability ⟨ψ|ψ⟩=1\langle \psi |\psi \rangle = 1. It appears that to get there, a should simply equal 1, but it appears there's a possibility that a= 1/i. So, right now my answer is a=1, because I sort of understand the wave function is necessarily a complex function.
The condition on a is ## |a|^2=a a^*=1 ## which is a circle in the complex plane. So there are an infinite number of solutions all equally valid. Two of them are 1 and 1/i.
squelch said:
d. The duel vector ⟨ψ|\langle \psi | is the complex conjugate of |ψ⟩|\psi \rangle . So, I might say that ⟨ψ|=13√|U⟩−a2√3√i|D⟩\langle \psi | = \frac{1}{{\sqrt 3 }}|U\rangle - \frac{{a\sqrt 2 }}{{\sqrt 3 }}i|D\rangle
Actually ## \langle \psi |= \frac{1}{\sqrt 3} \langle U |-\frac{a\sqrt 2}{\sqrt 3}i\langle D |##.
squelch said:
f. I feel like I have to make an assumption here: that it's fifty-fifty either way. If so, I might write: |ψ⟩=12√|R⟩−12√i|L⟩|\psi \rangle = \frac{1}{{\sqrt 2 }}|R\rangle - \frac{1}{{\sqrt 2 }}i|L\rangle
You should write up and down states in terms of right and left states and then substitute in ## |\psi\rangle ##. Its usually the case that right and left states are defined in terms of up and down states. So you should only solve two simultaneous algebraic equations to invert the definitions.
All other answers seem fine to me(except g and h which should change according to f).
 
  • #3
Thanks. I'll work on unpacking the correction you've given me for f.

Was the link to the lecture notes I provided a good reference to these sorts of problems? The professor threw us some stuff that wasn't in the textbook and I feel very much like I'm groping in the dark at my notes with this.
 
  • #4
I didn't check it carefully but it seemed good. Also I think you can trust MIT in such things! But I suggest you take a look at other notes of the same series too, just in case!
 
  • #5
I'm having a lot of trouble unpacking your last statement :
Shyan said:
You should write up and down states in terms of right and left states and then substitute in ## |\psi\rangle ##. Its usually the case that right and left states are defined in terms of up and down states.

It seems like you're suggesting I say [itex]|U\rangle = \frac{1}{{\sqrt 2 }}|L\rangle + \frac{1}{{\sqrt 2 }}|R\rangle [/itex] and [itex]|D\rangle = \frac{1}{{\sqrt 2 }}|L\rangle + \frac{1}{{\sqrt 2 }}|R\rangle [/itex] and say:

[tex]|\psi \rangle = \frac{1}{{\sqrt 3 }}(\frac{1}{{\sqrt 2 }}|L\rangle + \frac{1}{{\sqrt 2 }}|R\rangle ) + \frac{2ai}{{\sqrt 3 }}(\frac{1}{{\sqrt 2 }}|L\rangle + \frac{1}{{\sqrt 2 }}|R\rangle )[/tex]

Simplified as:

[tex]|\psi \rangle = \frac{1+2ai}{{\sqrt 6 }}|L\rangle + \frac{1+2ai}{{\sqrt 6 }}|R\rangle [/tex]

edit: corrected some algebra
 
Last edited:
  • #6
The way you wrote them, we have ## |U\rangle=|D\rangle ## which is wrong!
We know that we can write up and down states as linear combinations of right and left states and now we want to determine the coefficients. Let's call them ## a_u, b_u,a_d,b_d##, with the conditions ## |a_u|^2=|b_u|^2=|a_d|^2=|b_d|^2=\frac 1 2##. This condition comes from the fact that when the system is in a up or down eigenstate, we have no knowledge about its right-left condition and so they should be equally probable. Another condition on the coefficients is that they should give us orthogonal up and down states, so we have ## a_u a_d^*+b_u b_d^*=0##.
So we have four complex unknowns which means eight real unknowns. But the number of equations are six.(the norm equations are one equation each but the orthogonality equation has a real and a complex part.) So we have an underdetermined system which means we have some amount of freedom(two arbitrary real unknowns). What you should do, is first choosing two of the real unknowns and then use the above equations to determine the rest.
 

Related to Bra-Ket Notation, Wave Equation, Particle States

1. What is Bra-Ket Notation and how is it used in quantum mechanics?

Bra-Ket Notation, also known as Dirac Notation, is a mathematical notation used in quantum mechanics to represent quantum states and operations. It consists of the symbols < | and | >, which are called the bra and ket respectively, and an inner product < | > between them. The bra represents the dual vector to the ket, and the inner product represents the probability amplitude for measuring the state represented by the ket in the state represented by the bra.

2. What is the Wave Equation and how does it relate to quantum mechanics?

The Wave Equation is a partial differential equation that describes the propagation of waves. It is used in quantum mechanics to describe the behavior of quantum particles, as they exhibit both wave-like and particle-like properties. The Wave Equation is also known as the Schrödinger Equation and is a fundamental equation in quantum mechanics.

3. What are particle states in quantum mechanics?

Particle states in quantum mechanics refer to the set of quantum states that describe the properties of a particle, such as its position, momentum, and spin. These states are represented by wave functions, and their evolution over time is described by the Schrödinger Equation. The measurement of particle states is probabilistic, and the outcome of a measurement is determined by the wave function of the particle.

4. How do you represent a particle state using Bra-Ket Notation?

In Bra-Ket Notation, a particle state is represented by a ket | > which is a vector in a Hilbert space. This ket represents the state of the particle, and the bra < | represents the dual vector. The inner product < | > between the bra and ket gives the probability amplitude for measuring the particle in that state. By combining different kets and bras, one can represent complex quantum states and operations.

5. What is the significance of Bra-Ket Notation in quantum mechanics?

Bra-Ket Notation is a powerful tool in quantum mechanics as it allows for the representation of complex quantum states and operations in a concise and efficient manner. It also simplifies the mathematical calculations involved in quantum mechanics and provides a clear and intuitive understanding of the physical concepts. Bra-Ket Notation is widely used in quantum mechanics and has become an essential part of the language of modern physics.

Similar threads

  • Introductory Physics Homework Help
Replies
2
Views
364
  • Introductory Physics Homework Help
Replies
1
Views
720
  • Introductory Physics Homework Help
Replies
7
Views
1K
  • Introductory Physics Homework Help
Replies
5
Views
1K
  • Introductory Physics Homework Help
Replies
2
Views
940
  • Quantum Interpretations and Foundations
2
Replies
47
Views
2K
  • Introductory Physics Homework Help
Replies
9
Views
1K
  • Introductory Physics Homework Help
Replies
5
Views
819
Replies
16
Views
654
  • Introductory Physics Homework Help
Replies
1
Views
911
Back
Top